What is the slant height of the cone?

What Is The Slant Height Of The Cone?

Answers

Answer 1

Answer:

12.6

Step-by-step explanation:

Use pythag

8 / 2 = 4

a² + b² = c²

4² + 12² = c²

16 + 144 = c²

c² = 160

c = 12.6


Related Questions

The retail cost of a computer is 37% more than its wholesale cost?which statement is true?1. THe retail cost of the computer is 132% more than the wholesale price.2. THe wholesale cost of the computer is 68% if the retail price.3. THe retail cost of the computer is 132% of the wholesale price.4. the retail cost of the computer is 37% of the wholesale price.

Answers

when retail cost of the computer is 37% of the wholesale price then Retail price is 1.37times wholesale price

Retailers who acquire products in bulk are subject to wholesale pricing.

Selling products at a greater price than what it costs to produce them allows businesses to turn a profit.

Retail pricing is what merchants decide to charge customers as their ultimate selling price.

Consumers are the primary focus of retail pricing.

According to the question,

The retail cost of a computer is 37% more than its wholesale cost

Let Retail cost be "x" and wholesale cost be "y"

So , x = y + 0.37y

=> x = 1.37y

Therefore , The retail price is 1.37times the wholesale price

To know more about Retail price here

https://brainly.com/question/12929999

#SPJ4

Bobby and Rick are in a 16-lap race on a one-mile oval track. Bobby, averaging 86 mph, has completed six laps just as Rick is getting his car onto the track. What speed does Rick have to average to be even with Bobby at the end of
the sixteenth lap?
To be even with Bobby at the end of the sixteenth lap, Rick has to average a speed of __ mph.
(Type an integer or a decimal.)

Answers

Hence, the speed that Rick must be driving on to even with Bobby at the end of the 16th lap should be: Distance = Speed × Time16=1084× Speed Speed=16×8410=134.4 mph.

The time taken by Bobby to complete 10 laps will be:

[tex]$$\begin{aligned}\text { Distance } & =\text { Speed } \times \text { Time } \\10 & =84 \times \text { Time } \\\text { Time } & =\frac{10}{84} \text { hours }\end{aligned}$$[/tex]

Hence, the speed that Rick must be driving on to even with Bobby at the end of the 16 th lap should be:

[tex]$$\begin{aligned}\text { Distance } & =\text { Speed } \times \text { Time } \\16 & =\frac{10}{84} \times \text { Speed } \\\text { Speed } & =\frac{16 \times 84}{10}=134.4 \mathrm{mph} .\end{aligned}$$[/tex]

To more learn about speed visit:

https://brainly.com/question/28224010

#SPJ1

Ravi sells real estate. Based on previous data, he knows that 5% of home tours result in a sale. Assume that the results of these tours are independent from each other. Which of the following choices are binomial random variables? Choose all answers that apply: A. Take a random sample of 30 tours and let L = the number of tours that result in a sale. B. Take a random sample of 3 tours and let K = the number of tours that result in a sale. C. Take a random sample of 3 tours and let M = the amount of sales (in dollars) generated by the tours.

Answers

The options that represent binomial random variable  are;

A. Take a random sample of 30 tours and let L = the number of tours that result in a sale.

B. Take a random sample of 3 tours and let K = the number of tours that result in a sale.

How to Identify Binomial Random Variables?

There are 4 primary conditions for a random variable to be classified as binomial random variable and they are;

1. The number of observations n is fixed.

2. Each observation is independent.

3. Each observation represents one of two outcomes ("success" or "failure").

4. The probability of "success" p is the same for each outcome.

No, in this case, since 5% of home tours result in a sale, it therefore tells us that number of home tours is the independent variable while the amount of sales generated is the dependent variable.

From the above, we can access the given options and say that the last option is not a binomial random variable because the variable M is dependent and so does not satisfy one of the four conditions.

However, options A and B satisfy the 4 conditions and we say they are binomial random variables.

Read more about Binomial Random Variables at; https://brainly.com/question/15246027

#SPJ1

s formula to find a quadratic approximation of at the origin. estimate the error in the approximation if and .

Answers

A quadratic equation of at the origin, estimate the error = 0.000859M

The approximation is valid because  is very small.

Calculation of  concentration:

Since

0.85 M             0           0

(0.85-x)M         x            x

Now the value of x should be

x = 0.0000229

So based on this, the above concentration should be determined.

In order to demonstrate that the same value of x may be achieved either way, you will now solve using the quadratic formula rather than iterations. What are the values of a, b, and c and x, where a, b, and c are the coefficients in the quadratic equation [tex]ax^{2} +bx+c=0[/tex] and x is [h3o+], when using the quadratic equation to determine [h3o+] in 0.00250 m hno2? Keep in mind that ka=4.5104.

a : 1

b : 4.5x[tex]10^{-4}[/tex]

c : 1.125x[tex]10^{-6}[/tex]

[[tex]H_{3} O^{+}[/tex]] = 0.000859M

As [tex]HNO_{2}[/tex] is a weak acid, its equilibrium in water is:

[tex]HNO_{2} (aq)+H_{2} O(I)[/tex] ⇄ [tex]H_{3} O^{+} (aq)+N_{2} O^{-} (aq)[/tex]

Equilibrium constant, ka, is defined as:

ka = 4.5x[tex]10^{-4}[/tex] = [[tex]H_{3} O^{+}[/tex]]  [NO₂⁻] / [HNO₂]                    (Equation-1)

Equilibrium concentration of each specie are:

[HNO₂] = 0.00250M - x

[H₃O⁺] = x

[NO₂⁻] = x

Replacing in (1):

4.5x[tex]10^{-4}[/tex] = [tex]\frac{x*x}{0.00250M-x}[/tex]

1.125x10⁻⁶ - 4.5x10⁻⁴x = x²

0 = x² + 4.5x10⁻⁴x - 1.125x10⁻⁶

As the quadratic equation is ax² + bx + c = 0

Coefficients are:

a: 1

b: 4.5x10⁻⁴

c: 1.125x10⁻⁶

Now, solving quadratic equation:

x = -0.0013 → False answer, there is no negative concentrations.

x = 0.000859

As [H₃O⁺] = x; [H₃O⁺] = 0.000859M

Therefore,

A quadratic equation of at the origin, estimate the error = 0.000859M

To learn more about Quadratic equation visit :

brainly.com/question/17177510

#SPJ4

An area code has three digits. How many different area codes are possible

Answers

Answer:

1000

Step-by-step explanation:

If any of the digits 0-9 can be used then there are 10^3 possible codes.

10^3 = 1000

3. What type of angle is angle A?
obtuse
acute
right
straight

Answers

The angle A is an obtuse angle of the regular polygon.

What is an angle?

An angle is a figure in Euclidean geometry made up of two rays that share a common terminal and are referred to as the angle's sides and vertices, respectively. Angles created by two rays are in the plane where the rays are located. The meeting of two planes also creates angles. We refer to these as dihedral angles.

The interior angle of a polygon is [(n-2)180°]/n.

Where n is the number of sides of the polygon.

The number of sides of the polygon is 8.

The measure of the interior of the polygon is [(8-2)180°]/8

= 135°

The obtuse angle is an angle that more than 90°.

Therefore angle is an obtuse angle.

To learn more about interior angle, click on below link:

https://brainly.com/question/10566696

#SPJ1

Give expressions for the following:-
1) x is multiplied by -10 and then 5 is added to the result
2) 5 is multiplied by p and the result is subtracted from 26
3)y is multiplied by -5 and the result is added to 6

Answers

Answer:

1) -10x + 5

2) 5p - 26

3) -5y + 6

The gardening club at school is growing vegetables. The club has 300 square feet of planting
beds.Cucumber plants require 6 square feet of growing space, and tomato plants require 4
square feet of growing space. The students want to plant some of each type of plant and have at
least 60 plants.
Select the combination of equations or inequalities that could describe this situation. Let c represent the number of cucumber plants and t represent the number of tomato plants.

Answers

Answer:

The answer is :

E - 6c + 4t < 300

Point B is located at
(-3,0)
(3,0)
(0,3)
(0,-3)

Answers

Answer:

(3,0)

have a nice day.(I needed 20 charecters so yeah i needed to say this)

Answer:

(3,0)

Step-by-step explanation:

The first number in the ordered pair tells us how far we are from the orgina (0,0) (the middle of the graph).  We are 3 units to the right.  The second number in the ordered pair tells us how far up or down from (0,0) we are.  You do not go up for down so that number should be zero.

Find the least common denominator of these fractions.

3/8 5/18

Answers

Multiple 8 on denominator

Option C } 72

Mark Brainly

Answer: 72

Step-by-step explanation

Let's just look at the denominators for this problem.

8 and 18

8 can go into 24, 3 times but 18 does not multiply into 24

18 can go into 36, 2 times but 8 does not multiply into 36

Therefore 72 is the least common denominator.

8 times 9 = 72

18 times 4 = 72

72 is the least common denominator.

expresa 1405 en base 7 a base 10

Answers

1405 in base 7 = 544 in base 10

What is base-7 number system ?

The heptimal system is another name for base-7 numbers. Base 7 numbers are represented by the digits 0, 1, 2, 3, 4, 5, and 6.

The digits of a Base 10 number, on the other hand, are 0, 1, 2, 3, 4, 5, 6, 7, 8, and 9. If there isn't a subscript at the supplied number, Base 10 is used to write that number. The decimal system is sometimes known as base-10 numbers. Currently, in daily life, we frequently use numbers in the base 10 range.

base 7 to  base 10 conversion:

Multiply each digits by the powers of 7 as follows:

Base 7 digits:   1         4        0         5

Multiply by:      7^3      7²      7^1      7^0

  ( 1*7^3) + (4*7² ) + (0* 7^1 )+ ( 5 *   7^0)

= 343 + 196 + 0 + 5

= 544

To learn more about the base-7 number system from the given link

https://brainly.com/question/1674893

#SPJ1

Amy and three friends are renting a house near the campas for $2400 per month. She plays 1/4 of the rent. What is her monthly rent?

Answers

Answer:

Amy has to pay $600 every month for rent.

Step-by-step explanation:

To take 1/4 of a number is to multiply 1/4 and said number.

So you can do 0.25*number

or number/4.

For this problem, the number is 2400 because Amy is going to pay 1/4 of 2400 for rent.

[tex]rent = 2400/4\\rent = 600[/tex]

⭐if this response helped you, please mark it the "brainliest"!⭐

the height of one tampa city center is 537 feet. convert 537 feet to meters by finding an equivalent rate. round to the nearest tenth

Answers

The equivalent rate is 163.7 meters.

What is an equivalent rate?

Equivalent rates are different rates that have the same value. Similar to finding equivalent ratios, you may find an equivalent rate by multiplying or dividing the numerator and denominator by the same number.

Here, we have

Given: the height of one Tampa city center is 537 feet.

We have to convert 537 feet to meters by finding an equivalent rate.

1 feet = 0.3048 meter

So, 537 feet = 537 × 0.3048 = 163.7 meters.

Hence, the equivalent rate is 163.7 meters.

To learn more about the equivalent rate from the given link

https://brainly.com/question/24734894

#SPJ1

Answer:

Hence, on a number line,-12 lies to the left of Zero.

Step-by-step explanation:

On a number line, zero lies exactly at the middle of the number line.Left to the zero, negative number lies and right to the zero, positive number lies.

Please Help me!!!!! Will give brainliest 4 an EXPLAINATION!

Answers

The angles after solving the equations will be equal to 50°, and both angles will be the same as the corresponding angles. Hence, option B is correct.

What is an angle?

An angle results from the intersection of two lines at a point. The term "angle" describes the width of the "gap" that exists between these two rays. It's represented by the symbol.

Angles are most frequently measured in degrees and radians, a measurement of roundness or rotation. Angles are a part of everyday existence.

As per the given information in the question,

The equations for the angles are:

7x + 1 = 6x + 8

7x - 6x = 8 - 1

x = 7

So, the angles will be,

7x + 1 = 7(7) + 1 = 50°

6x + 8 = 6(7) + 8 = 50°

To know more about an angle:

https://brainly.com/question/28451077

#SPJ1

in the morning there were t people at the beach. later at noon, 2/7 of the people left the beach and there were 30 people left on the beach. find t

Answers

Answer:

42

Step-by-step explanation:

If 2/7 left that means that 5/7 are still there.

5/7x = 30  Multiple both sides by 7/5

x = [tex]\frac{30}{1}[/tex] x [tex]\frac{7}{5}[/tex]

x = 42

whats the domain,range, and y-intercept and tell me if its exponential or linear.

k(x)=50(1.3)^x

Answers

0.3% is the percentage rate of increase  in exponential function.

What exactly makes a function exponential?

A mathematical function using the following formula is an exponential function: f (x) = an x. where an is a constant known as the function's base and x is a variable.

                   The transcendental number e, or roughly 2.71828, is the most often encountered exponential-function base.

We have,

y = 50( 1.3)ˣ

The equation represents exponential growth because the growth factor is greater than 1.

The general form equation is

y(x)= a(1-r)^x such that r is the growth percent.

   1 + r = 1.3

     r = 0.3 ⇒  0.3%

Learn more about exponential function

brainly.com/question/14355665

#SPJ1

93 is what percent of 124

Answers

Answer: 75%

Step-by-step explanation:



93 is 75 percent of 124.

Explained in the picture attached...

Hope that helps...

Can I get some help with this?

Answers

The equations that represent given problem is

P + S = 384 and 2.50P + 1.25S = 696.25.

What is an equation?

A mathematical statement known as an equation is made up of two expressions joined together by the equal sign. A formula would be 3x - 5 = 16, for instance. When this equation is solved, we discover that the value of the variable x is 7.

Given that the cost of 1 slice pizza is $2.50.

The cost of 1 bottle can of soda is $1.25.

In total sell for a day is 384.

Assume that you sell P slice of pizza and S number of soda bottle.

Thus your total sell is P + S.

Therefore, P + S = 384.

The cost 1 slice pizza is $2.50

Then the cost P slice of pizza is $2.50P.

The unit rate of soda bottle is  $1.25.

Then the cost S bottles of soda is $1.25S.

Therefore the total sell for that day is 2.50P + 1.25S

2.50P + 1.25S = 696.25

To learn more about system of equations, click on below link:

https://brainly.com/question/13973975

#SPJ1

Find the area of a rectangle with the sides of x+7 and 3x-4

Answers

The area of a rectangle with the sides of x+7 and 3x-4 is 3x² + 17x - 28.

Define area of rectangle.

Any shape's area can be calculated by counting how many unit squares will fit inside of it. A square with a side of 1 unit is referred to as a unit square in this context. Therefore, the quantity of unit squares that make up a rectangle's perimeter is its area. Alternately, the area of the rectangle is the area contained within the boundary of this shape. The unit-length tiles in your home are a good illustration of a rectangle form. By counting the number of tiles, you can quickly determine how much space the floor takes up. This will also enable you to calculate the rectangle floor's area.

Given

Sides = x+7 and 3x-4

Area of rectangle

Length ×Breadth

(x + 7) (3x - 4)

Multiplying,

x(3x - 4) + 7(3x - 4)

3x² - 4x + 21x - 28

3x² + 17x - 28

The area of a rectangle with the sides of x+7 and 3x-4 is 3x² + 17x - 28.

To learn more about area of rectangle, visit:

https://brainly.com/question/20693059

#SPJ1

Use Figure 1 to evaluate the trigonometric function.

A right triangle with side a of length 13 opposite of angle A, and side b of length 6 opposite of angle B.136
Figure 1
Enter the exact answer.



tanB=







Change entry mode


Show your work and explain, in your own words, how you arrived at your answer. Answers with no relevant explanations may receive reduced or no credit.




Answers

third side,  AB= 14.318 Units  and  tanB = 0.461

How to find the value of tanB and third side?

given:

BC=13 units

AC=6 units

solution:

using pythagoras theorem, we get

AB² = BC² + AC²

AB² = 13² + 6²

AB² = 169 + 36

AB² = 205

AB = 14.318 units

tanB = side opposite/side adjacent

tanB = AC/BC

tanB = 6/13

tanB = 0.461

to know more about Pythagoras theorem, visit

https://brainly.com/question/343682

#SPJ1

Using Pythagoras theorem, Third side,  AB= 14.318 Units  and tanB = 0.461

What is the Pythagoras theorem theory?

The Pythagorean Theorem states that the squares on the hypotenuse (the side across from the right angle) of a right triangle, or, in standard algebraic notation, a2 + b2, are equal to the squares on the legs.

How to find the value of tanB and third side?

given:

BC=13 units

AC=6 units

solution:

using pythagoras theorem, we get

AB² = BC² + AC²

AB² = 13² + 6²

AB² = 169 + 36

AB² = 205

AB = 14.318 units

tanB = side opposite/side adjacent

tanB = AC/BC

tanB = 6/13

tanB = 0.461

to know more about Pythagoras theorem, visit

brainly.com/question/343682

#SPJ1

MULTIPLE CHOICE Question 2 Look at the drawing below. Which of the following statements, if true, would alone prove pllq? Select all that apply. There are at least two correct answers.​

Answers

Answer:

no c is right answer

Step-by-step explanation:

angle b and e should be supplementary

Answer:

A, B and C

Step-by-step explanation:

Statement A

According to the Alternate Exterior Angles Theorem, when a transversal line intersects two parallel lines, the resulting alternate exterior angles are congruent.

Given ∠c ≅ ∠f, and as ∠c and ∠f are alternate exterior angles, then p ║ q.

Statement B

According to the Corresponding Angles Postulate, when a straight line intersects two parallel straight lines, the resulting corresponding angles are congruent.

GIven ∠d ≅ ∠g, and as ∠d and ∠g are corresponding angles, then p ║ q.

Statement C

According to the Same-side Interior Angles Theorem, when two parallel lines are intersected by a transversal, the angles that are interior to the parallel lines and on the same side of the transversal line are supplementary.

Given ∠b and ∠e are supplementary, and as ∠b and ∠e are the interior angles on the same side of the transversal [tex]\ell[/tex], then p ║ q.

Statement D

∠f and ∠g are a linear pair, therefore ∠f and ∠g are supplementary.  However, this does not given enough information to prove p ║ q.

Which quantity is multiplied by pi in the formula for the area of a circle?

Answers

The quantity that is multiplied by pi in the formula for the area of a circle is the square of the radius

How to determine the multiplied quantity?

From the question, we have the following parameters that can be used in our computation:

Area of a circle

The formula for the area of a circle is represented as

A = πr²

Express the formula as a product

So, we have the following representation

A = π * r²

In the above formula, we can see that the square of r is multiplied by pi

Hence, the required quantity is the square of the radius of the circle

Read more about circle area at

https://brainly.com/question/10645610

#SPJ1

Solve the equation 2x + 3y = 5 for x.

Answers

Answer:

x = [tex]\frac{5-3y}{2}[/tex]

Step-by-step explanation:

2x + 3y = 5

isolate variable: 2x = 5-3y

divide by 2: x = [tex]\frac{5-3y}{2}[/tex]

The parabola of y= has a vertex of (3, - 2) and a focus of (3, - 2 1/16) opens downward

Answers

The equation of the parabola is y = -1/4 x² + 3/2 x - 17/4. Where the vertex of the equation is at (3, -2) and the focus is at (3, -2 2/16) that opens downwards.

What is the equation of a parabola?

The equation of the parabola with vertex at (h, k) is

y = a(x - h)² + k

The focus of the parabola is represented by (h, k + 1/4 a).

Calculation:

It is given that, the vertex of the parabola is (h, k) = (3, -2)

And the focus of the parabola is (h, k + 1/4 a) = (3, -2 1/16)

From the focus point, we can calculate the value of 'a'.

(h, k + 1/4 a) = (3, -2 1/16)

⇒ k + 1/4 a = -2 1/16 = -33/16

⇒ -2 + 1/4 a = -33/16

⇒ 1/4 a = -33/16 + 2

⇒ 1/4 a = -1/16

⇒ a = -1/16 × 4

∴ a = -1/4

Since a is negative the parabola is downwards.

Now, the equation of the parabola is

y = a(x - h)² + k

On substituting the values, we get

y = -1/4(x - 3)² - 2

  = -1/4(x² - 6x + 9) - 2

  = -1/4 x² + 3/2 x -9/4 - 2

  = -1/4 x² + 3/2 x - 17/4

Therefore, the equation of the parabola is y = -1/4 x² + 3/2 x - 17/4.

Learn more about the equation of a parabola here:

https://brainly.com/question/12464740

#SPJ1

A punter kicks a football. Its height h, in yard, t seconds after the kick is given by the equation h(t)=-4.9t^2+18.24t+0.8. The height of an approaching blocker's hand is modeled by the equation h(t)=-1.43t+4.26, using the same time. Can the blocker knock down the punt (do they intersect)? If so, at what point will that happen (the point of intersection)?

Answers

Part 1

[tex]-4.9t^2 +18.24t+0.8=-1.43t+4.26\\\\-4.9t^2 +19.67t-3.46=0\\\\\Delta =(19.67)^2 -4(-4.9)(-3.46)=319.0929 > 0[/tex]

Therefore, the blocker can knock down the punt.

Part 2

Using the quadratic formula,

[tex]t=\frac{-19.67 \pm \sqrt{319.0929}}{2(-4.9)}\\\\t \approx 0.18437, 3.82992[/tex]

Considering the graphs, it is clear to take the smaller solution. Thus, the point of intersection is [tex](0.18437, h(0.18437))=\boxed{(0.18437, 3.99635)}[/tex].

Status
Review
A certain state uses the following progressive
tax rate for calculating individual income tax:
Income
Range ($)
0 - 10,000
10,001 - 50,000
50,001 - 100,000
Progressive
Tax Rate
3%
5%
5.5%
Calculate the state income tax owed on a $50,000
per year salary.
tax = $[?]
Round your answer to the nearest whole dollar amount.
Enter
h

Answers

Answer:

final answer would be $6,550

Step-by-step explanation:

To calculate the state income tax owed on a $50,000 per year salary, we can use the progressive tax rate provided in the question. We can divide the salary into three ranges, corresponding to the three tax rates:

Income Range: $0 - $10,000

Progressive Tax Rate: 3%

Tax Owed: $0 - $10,000 * 3% = $0 - $300

Income Range: $10,001 - $50,000

Progressive Tax Rate: 5%

Tax Owed: $10,001 - $50,000 * 5% = $500 - $2,500

Income Range: $50,001 - $100,000

Progressive Tax Rate: 5.5%

Tax Owed: $50,001 - $100,000 * 5.5% = $2,750 - $5,500

Thus, the total state income tax owed on a $50,000 per year salary would be $300 + $2,500 + $2,750 = $6,550. This amount should be rounded to the nearest whole dollar amount, so the final answer would be $6,550.

A man is in a boat 2 miles from the nearest point on the coast. He is to go to point Q, located 3 miles down the coast and 1 mile inland (see figure). He can row at a rate of 1 mile per hour and walk at 3 miles per hour. Toward what point on the coast should he row in order to reach point Q in the least time? (Round your answer to two decimal places.) 0.84 mile(s) down the coast

Answers

Least time required to reach the point Q as per the distance and the speed rate is equal to 2 hours.

As given in the question,

Nearest point on the coast is 2 miles far away

rate of the row = 1mile per hour

Walk at the rate of 3 miles per hour

Let 'x' hours be the least time to reach point Q.

Time = distance / speed

Time taken to reach the point Q = [√ 1 + ( 3 - x)² ]/ 3

Time taken to reach the coast = (√ 4 + x² ) / 1

Total  time taken 't' = (√ 4 + x² ) / 1 + [√ 1 + ( 3 - x)² ]/ 3

To find least time dt/dx = 0

t  = (√ 4 + x² ) / 1 + [√ 1 + ( 3 - x)² ]/ 3

⇒dt/dx = [ x / √ 4 + x² ] + ( 3 - x) / √( 10 -6x + x² )

⇒x / √ 4 + x² = ( x - 3) / √( 10 -6x + x² )

Squaring both the side we get,

x² / (4 + x²) = ( x - 3)² / ( 10 -6x + x² )

⇒3x² -24x +36 =0

⇒ x² -8x + 12 = 0

⇒ x = 2 or 6 hours

Therefore , the least time taken to reach the point Q is equal to 2 hours.

Learn more about least time here

brainly.com/question/17429465

#SPJ4


Examine the drawing below, which could be a value for x?

Answers

The value of 'a' can be 20. The correct option is C, 20.

What is triangle inequality theorem?

As per the triangle inequality theorem, the sum of any two sides of the triangle should be greater than the third side.


As per triangle inequality theorem, the sum of the two sides of the triangle should be greater than the third side of the triangle.

As per the triangle inequality theorem, if 'a' is the longest side of the triangle then,

21 + 6 > a

27 > a

If a is not the longest side of the triangle,

21 + a > 6

a > -21 + 6

a > -15

6 + a  > 21

a > 21 - 6

a > 15

Therefore, the value of a should be greater than 15 and less than 27. Since the only option under this condition is 20.

Hence, The correct option is C, 20.

Learn more about the Triangle Inequality Theorem here:

https://brainly.com/question/1163433

#SPJ1

Is there anyone that can help me with a finance question?

Answers

Answer:

Yes, there are many people who can help you with a finance question. Some of the people who can help you include: financial advisors, accountants, financial planners, and financial analysts. Additionally, there are many online resources available such as personal finance forums, websites, and blogs.

Step-by-step explanation:

Check the binomial distribution to see whether it can be approximated by the normal distribution. Round p and q to 1 decimal place, as needed. n = 95 P = 0.96 9 -0.04 np - and ng Is a normal approximation appropriate ? Yes No

Answers

As per the binomial distribution, the value of the normal approximation is 0.6573

The term binomial distribution refers the discrete probability distribution that gives only two possible results in an experiment, either success or failure.

Here we have given that n = 95 P = 0.96 and q = 0.04

Now, here we have to check  the binomial distribution to see whether it can be approximated by the normal distribution.

While we looking into the given question we know that the value of n = 95 P = 0.96.

Then as per the binomial distribution formula, the normal distribution is calculated as,

=> P(X=1) = 95C4 * (0.96)⁴ * (1-0.96)⁹⁵⁻⁴

When we simplify this one then we get the value as 0.6573

To know more about Binomial distribution here.

https://brainly.com/question/14565246

#SPJ4

Other Questions
suppose that the genome of an organism is 23% adenine (a). what should be true about the organism's genome? Diffusion patterns have been influenced by the growth of modern telecommunications. Which form of diffusion is affected LEAST by modern communication methods? Fill in the missing values in the following table, selecting the answers closest to the values you calculate.YearQuantity of MoneyVelocity of MoneyPrice LevelQuantity of OutputNominal GDP(Dollars)(Dollars)(Cell phones)(Dollars)2017200??5.00400?201820210??400??The money supply grew at a rate of (.5%, 1%, 1.25%, 101%) from 2017 to 2018. Since cell phone output did not change from 2017 to 2018 and the velocity of money (increased, decreased, remained the same), the change in the money supply was reflected (partially, entirely) in changes in the price level. The inflation rate from 2017 to 2018 was (.5%,1%,1.25%,101%). The Payment for Environmental Services Act (PSA) was designed to protect forests and their services. How did Costa Rica fund the PSA? compare and contrast homosexual parenting to single parenting, teen parenting OR grandparents raising their grandchildren (pick one). What issues do homosexual parents have in common with the group you selected? Do you think the the process of mothering or fathering is the same for the two groups or different and why? What issues do homosexual parents have in common with heterosexual parents? How has the first trip changed Dana? Which set of factors corresponds to a third-degree polynomial with rational coefficients that has zeros x=2 and x=3i Ren runs a hamburger stand and needs to clean up a seasons worth of grease buildup on the range hood over his griddle. Explain what he should look for in a cleaning product that can handle a layer of grease. what is this Calculate the number of moles of carbon dioxide gas if the volume is 2.0 L, the pressure is 2.0 atm, and the temperatue is 410 K.Use three significant figures in your answer.mol A game uses an unbiased die with faces numbered 1 to 6. The die is thrown once. If it shows an even number this number is doubled to get the final score. If an odd number is thrown then the die is thrown again and the final score is the sum of the numbers shown on the two throws.Given that the dice was thrown twice. What is the probability that the final score is 8 if it requires energy u to accelerate a rocket fro rest to 1/2c the energy needed to acclerate a rocket from 1/2 c to c would be When you pass an array as a parameter, the base address of the actual array is passed to the formal parameter. t/f Question 1 of 7 Question 1>The relationship between the amount of time a car is parked, in hours, and the cost of parking, in dollars, can be described with a function. Whichstatements about the function are true? Select all that apply.A HELP ASAP!how culture, economics, and politics had on Jacob van Ruisdael art What caused the United States to become involved in World War I, and how did the United States change as a result of its involvement? Answer this question in two paragraphs. Each paragraph should have a topic sentence and a minimum of 5-6 sentences. HELP ME PLEASE THIS IS A LAST MINUTE ASSIGNMENT!! What are the chances of 1%-100% of my mom letting me play video games if I stole her credit card to buy V-bucks and not doing the house chores?Why or why not? Explain. Doctors have a legal right to treat patients by any method that will preserve quality of life, even if the patient wishes not to undergo treatment.OA. TrueOB. False True or False: The Salk polio vaccine (IPV) is considered to be much safer but less effective than the Sabin polio vaccine (OPV). asnwer ASAP RIGHT NOW A triangle has side lengths of (9v-7w)(9v7w) centimeters, (7v+10x)(7v+10x) centimeters, and (8x+5w)(8x+5w) centimeters. Which expression represents the perimeter, in centimeters, of the triangle?